# Magic with Powers of a Matrix
This post will use powers of a matrix to calculate recursively-defined functions, and then use the eigendecomposition to find closed form solutions. We'll start off with the Fibonacci numbers and move on to other functions, I guess, when I have time to post more :). ### Recap of the Eigendecomposition and Powers of a Matrix If a square \\( n \times n \\) matrix \\( A \\) has \\( n \\) linearly independent eigenvectors, it can be factorized as $$ A = Q \Lambda Q^{-1} $$ where \\(Q\\) is a square matrix with the eigenvectors as columns, and \\( \Lambda \\) is a diagonal matrix with the corresponding eigenvalues on the diagonal. This is called the eigendecomposition of a matrix. Now consider the powers of the matrix \\( A \\). Lets start with \\( A^2 \\), \begin{align} A^2 &= AA \\\\ &= (Q \Lambda Q^{-1})(Q \Lambda Q^{-1}) \\\\ &= Q \Lambda (Q^{-1} Q) \Lambda Q^{-1} \\\\ &= Q \Lambda I \Lambda Q^{-1} \\\\ &= Q \Lambda^2 Q^{-1} \end{align} By induction, we get the same cancellation for higher (positive integer) powers, giving us the incredible $$ A^n = Q \Lambda^n Q^{-1} $$ ## Fibonacci Numbers with Powers of a Matrix The Fibonacci numbers form a recursively defined sequence, with \\( F\_0=0, F\_1 = 1 \\) as base cases and \\( F\_n = F\_{n-1} + F\_{n-2} \\) for \\(n>1\\). I'll claim the powers of this matrix give us the Fibonacci numbers, $$ \\begin{bmatrix} 1 & 1 \\\\ 1 & 0 \\end{bmatrix}^n = \\begin{bmatrix} F\_{n+1} & F\_{n} \\\\ F\_{n} & F\_{n-1} \\end{bmatrix} $$ #### Proof We'll do this by induction. The base case is trivial, $$ \\begin{bmatrix} 1 & 1 \\\\ 1 & 0 \\end{bmatrix}^1 = \\begin{bmatrix} F\_{2} & F\_{1} \\\\ F\_{1} & F\_{0} \\end{bmatrix} $$ The inductive step is also pretty easy, \begin{align} \\begin{bmatrix} 1 & 1 \\\\ 1 & 0 \\end{bmatrix}^n &= \\begin{bmatrix} 1 & 1 \\\\ 1 & 0 \\end{bmatrix}^{n-1} \\begin{bmatrix} 1 & 1 \\\\ 1 & 0 \\end{bmatrix} \\\\ &= \\begin{bmatrix} F\_{n} & F\_{n-1} \\\\ F\_{n-1} & F\_{n-2} \\end{bmatrix} \\begin{bmatrix} 1 & 1 \\\\ 1 & 0 \\end{bmatrix} && \\text{(inductive hypothesis)} \\\\ &= \\begin{bmatrix} F\_{n} + F\_{n-1} & F\_{n} \\\\ F\_{n-1} + F\_{n-2} & F\_{n-1} \\end{bmatrix} \\\\ &= \\begin{bmatrix} F\_{n+1} & F\_{n} \\\\ F\_{n} & F\_{n-1} \\end{bmatrix} && \blacksquare \end{align} #### Applying the Eigendecomposition Taking the eigendecomposition of the matrix \\( \\begin{bmatrix} 1 & 1 \\\\ 1 & 0 \\end{bmatrix} \\), we get \begin{align} Q &= \\begin{bmatrix} \varphi & -\varphi^{-1} \\\\ 1 & 1 \\end{bmatrix} \\\\ \Lambda &= \\begin{bmatrix} \varphi & 0 \\\\ 0 & -\varphi^{-1} \\end{bmatrix} \\\\ Q^{-1} &= \frac{1}{\varphi + \varphi^{-1}} \\begin{bmatrix} 1 & \varphi^{-1} \\\\ -1 & \varphi \\end{bmatrix} \\\\ \end{align} Where \\( \varphi = (1 + \sqrt{5})/2 = 1.61803\dots \\) is the golden ratio. Notice the eigenvalues turn out to be the golden ratios (there are two golden ratios, but I digress). There is certainly something very special about this matrix. Now we know that we can calculate the \\( n^{th} \\) Fibonacci number as \begin{align} F_n &= \begin{bmatrix} 0 & 1 \end{bmatrix} \\begin{bmatrix} F\_{n+1} & F\_{n} \\\\ F\_{n} & F\_{n-1} \\end{bmatrix} \begin{bmatrix} 1 \\\\ 0 \end{bmatrix} \\\\ &= \begin{bmatrix} 0 & 1 \end{bmatrix} \\begin{bmatrix} 1 & 1 \\\\ 1 & 0 \\end{bmatrix}^n \begin{bmatrix} 1 \\\\ 0 \end{bmatrix} \\\\ &= \begin{bmatrix} 0 & 1 \end{bmatrix} Q \Lambda^n Q^{-1} \begin{bmatrix} 1 \\\\ 0 \end{bmatrix} \\\\ &= \begin{bmatrix} 1 & 1 \end{bmatrix} \\begin{bmatrix} \varphi^n & 0 \\\\ 0 & (-\varphi^{-1})^n \\end{bmatrix} \frac{1}{\varphi + \varphi^{-1}} \\begin{bmatrix} 1 & \varphi^{-1} \\\\ -1 & \varphi \\end{bmatrix} \begin{bmatrix} 1 \\\\ 0 \end{bmatrix} \\\\ &= \frac{1}{\varphi + \varphi^{-1}} \begin{bmatrix} \varphi^n & (-\varphi^{-1})^n \end{bmatrix} \begin{bmatrix} 1 \\\\ -1 \end{bmatrix} \\\\ &= \frac{\varphi^n - (-\varphi^{-1})^n}{\varphi + \varphi^{-1}} \\\\ &= \frac{1.618\dots^n - (-0.618\dots)^n}{2.236 \dots} \end{align} Which is pretty cool, we got a non-recursive closed form! One corollary here is that \\( F\_n = \frac{1.618\dots^n}{2.236 \dots} + O(0.618 \dots^n) \approx \frac{1.618\dots^n}{2.236 \dots} \\) for large enough \\( n \\), which means \\( F\_n = \Theta(\varphi^n) \\). We'll generally find that the asymptotics will be determined by the spectrum \\( \Lambda \\), which makes a lot of sense when looking at \\( A^n = Q \Lambda^n Q^{-1} \\).